Search type Search syntax
Tags [tag]
Exact "words here"
Author user:1234
user:me (yours)
Score score:3 (3+)
score:0 (none)
Answers answers:3 (3+)
answers:0 (none)
isaccepted:yes
hasaccepted:no
inquestion:1234
Views views:250
Code code:"if (foo != bar)"
Sections title:apples
body:"apples oranges"
URL url:"*.example.com"
Saves in:saves
Status closed:yes
duplicate:no
migrated:no
wiki:no
Types is:question
is:answer
Exclude -[tag]
-apples
For more details on advanced search visit our help page
Results tagged with
Search options answers only not deleted user 7076

Prime numbers, diophantine equations, diophantine approximations, analytic or algebraic number theory, arithmetic geometry, Galois theory, transcendental number theory, continued fractions

2 votes

Asymptotic of $Lcm(\binom{2k}k)_{1\le k\le n}$

Kummer's theorem implies the formula: $$\mathrm{LCM}(\binom{2}{1},\binom{4}{2},\dots,\binom{2n}{n}) = 2^{\lfloor \log_2(n+1)\rfloor} \cdot \prod_{3\leq p\leq 2n} p^{\lfloor \log_p(2n-1)\rfloor},$$ whe …
Max Alekseyev's user avatar
2 votes
Accepted

Characters of integers of $4p^2q^2-(p\pm q)^2$ form

It can be seen that both forms can be factored as differences of squares. Correspondingly, $n$ can be expressed in these forms iff there exists a divisor $d\mid n$ such that $d$ and $\frac{n}{d}$ have …
Max Alekseyev's user avatar
2 votes

Squares of the form $2^j\cdot 3^k+1$

It is also possible to reduce to 9 Mordell equations. Denoting $x := 2^{[i/3]}3^{[k/3]}$, we get 9 equations: $$d x^3 + 1 = y^2$$ indexed by $d\mid 2^23^2$. Equivalently, $$(dy)^2 = (dx)^3 + d^2,$$ wh …
Max Alekseyev's user avatar
8 votes

Partitioning $2p$, subject to a divisibility condition.

It is possible. Take $p=61$ and $t_1=2$, $t_{120}=1$. Then $l=113$ and $1^{t_1}\cdot t_1!\cdot 120^{t_{120}}\cdot t_{120}!=240$ divides $(2p-l)!=9!=362880$. Another example: $p=673$, $t_1=t_{672}=2$. …
Max Alekseyev's user avatar
4 votes

s(n) = kn or s(n) = n/k?

This is related to some open questions in number theory. See http://oeis.org/A134639 and references/links there. Conjecture 2 is incorrect, the smallest counterexample is given by $$n=154345556085770 …
Max Alekseyev's user avatar
3 votes

Need to require a assertion for two subsets of natural numbers

Consider the set $$T = \{ (x,y)\ :\ 1\leq x\leq Q,\ 1\leq y\leq P,\ Px+Qy\leq PQ\}.$$ It is easy to see that $$S = \sum_{(x,y)\in T} (Px+Qy).$$ Switching to double summation in $S$, we get $$S = \sum …
Max Alekseyev's user avatar
4 votes

Is there an asymptotic formula that describes the correlation of multiplicative inverses in ...

Finding asymptotic for $S(X)$ amounts to estimation of $\sum_{(a,q)=1} aa^*$ for any given $q$. From the rearrangement inequality, it follows that $$\frac{1}{2}q^2\varphi(q) - T = \sum_{(a,q)=1} a(q …
Max Alekseyev's user avatar
1 vote

Euler's theorem for Tetration, Pentation, etc. (Superexponentiation)

Suppose that for a positive integer $m$, there exists a positive integer $a>1$ such that $(a,m)=1$ and $\mathrm{rad}(\mathrm{ord}_m(a))\not|a$. Then $f_2(m)>0$ does not exists. (Here $\mathrm{ord}_m( …
Max Alekseyev's user avatar
4 votes
Accepted

Restriction of Jacobi's four-squares theorem

By inclusion-exclusion principle, the number of representations of $n$ as the sum of squares of four nonzero integers equals: $$\sum_{k=0}^4 \binom4k (-1)^k r_{4-k}(n).$$ Formulae for $r_k(n)$ are giv …
Max Alekseyev's user avatar
4 votes

how can I minimise (n * y) (mod x) for known x and y, and for a given range of n?

What I'm going to say is somewhat similar to Roland's answer but more precise in the case when the range for $n$ is given in the form of upper bound, i.e., $0 < n < N$. Notice that $ny\bmod x = ny - …
Max Alekseyev's user avatar
3 votes

Solutions of the equation $2^{q-1} \equiv q \pmod {4q^2+1 }$ where $q$ is an odd prime

I've checked all primes $q$ below $10^{11}$ with no new solutions found (even if the primality of $p=4q^2+1$ is relaxed). Notice that $2^{q-1}\equiv q\pmod{4q^2+1}$ implies $2^{2q}\equiv 4q^2\equiv - …
Max Alekseyev's user avatar
2 votes

A sum involving mod(n) arithmetic

I believe the formula $|A|=\frac{\varphi(q)}{\mathrm{lcm}(\varphi(d_1),\varphi(d_2))}$ is not quite correct. In particular, for $q=15$, $d_1 = 3$, $d_2=5$, $a=1$, this formula gives $|A|=2$, while, in …
Max Alekseyev's user avatar
8 votes

Elementary divisibility problem

Zsigmondy theorem states that for any $p>1$, $a^p+b^p$ has a primitive prime factor, except when $\{a,b\}=\{1,2\}$. Such prime does not divide $a+b$. Hence, $a^p+b^p$ cannot divide $(a+b)^p$ unless it …
Max Alekseyev's user avatar
1 vote

The existence of solutions of a system of indeterminate equations

I expect that in many cases the following construction will do the job. By Fermat polygonal number theorem, we have representation of $\frac{m(m-3)}4$ as the sum of 3 triangular numbers: $$ \frac{m(m- …
Max Alekseyev's user avatar
2 votes
Accepted

The existence of solutions of a system of indeterminate equations

In the comments OP proposed a greedy algorithm to represent a given positive integer $A$ as the sum of triangular numbers whose indices sum to $m$, and applied it to $A = \frac{m(m+1)}4$. I will prove …
Max Alekseyev's user avatar

1
2 3 4 5
16
15 30 50 per page